Machen Mechs überhaupt Sinn? - SciFi-Forum

Ankündigung

Einklappen
Keine Ankündigung bisher.

Machen Mechs überhaupt Sinn?

Einklappen
X
 
  • Filter
  • Zeit
  • Anzeigen
Alles löschen
neue Beiträge

    Das habe ich mit dem Impulserhaltungssatz gerechnet. Den laut tbfm2
    Zitat von tbfm2 Beitrag anzeigen
    @nightwalker:
    Hör doch bitte auf mit dieser dämlichen Unterstellung, jedes Grundschulkind kennt das Prinzip der Impulserhaltung. Auch möchte ich mal gerne wissen, warum du zum einen alles auf mich beziehst und zum anderen, warum du glaubst, ich würde den Impulserhaltungssatz nicht kennen?
    Nun ich habe einen bewusst einen leichten Mech Genomen bei den es möglich wäre so einen Treffer einzustecken (wäre aber unglaublich teuer) Damit überhaupt eine „merkliche“ Impulsänderung da ist.
    Jolly ich habe nichts von rammen gesagt lese es doch einfach noch mal

    Kommentar


      Klingt erstmal nett, kannst du auch die Berechnungen dafür posten? Wäre nämlich schön zu sehen, wie genau du auf diese Werte gekommen bist Bin grade zu faul, mir die Geschoss- und Kanonenwerte von überall zusammen zu suchen, ausserdem wäre es ja auch interessant zu sehen, wo du den Treffer angesetzt hast, (wegen Schwerpunkt usw.) und wie die Masseverteilung im Mecha bei dieser Berechnung ausgesehen hat.

      Aber ich habe die Befürchtung, das die Panzerung eben nicht hält *g*
      Jede Geschichte hat vier Seiten: Deine Seite, Ihre Seite, die Wahrheit und das, was wirklich geschehen ist.

      Welten brechen auseinander, Formationen nicht.

      Kommentar


        Zitat von nightwalker Beitrag anzeigen
        Hy Suthriel
        Wen ein Leopart2 einen 20t Mech mit einen DM63 trieft und die Panzerung hält wird sein Geschwindigkeit nach vorne um 0,696 Meter/s reduziert. Was wen er sich mit 14Meter/s bewegt kaum einen Einfluss hat. Wen er Steht und der Ausstemreflex ganze 0,4s dauert (was ultralangsam wäre) hätte er sich um 27,8 cm nach hinten bewegt was nicht gerade beeindrucken ist.
        Ich ERWARTE jetzt von dir das du eine Quelle nennst (Belegst) aus der diese zahlen stammen und wie du auf deine Berechnung kommst..
        ( ich habe die relevanten zahlen,der Mündungsgeschwindigkeiten und Geschossgewichte des Leo 2 parat (mit Link) und bin in der Lage deine Berechnungen zu überprüfen) also überasch uns mal mit FUNDIERTEM wissen (wenn du das kannst)
        .>ACHTUNG, freilaufender "Linker Gutmensch"! VORSICHT BISSIG!<

        Kommentar


          Zitat von nightwalker Beitrag anzeigen
          Das habe ich mit dem Impulserhaltungssatz gerechnet. Den laut tbfm2

          Nun ich habe einen bewusst einen leichten Mech Genomen bei den es möglich wäre so einen Treffer einzustecken (wäre aber unglaublich teuer) Damit überhaupt eine „merkliche“ Impulsänderung da ist.
          Jolly ich habe nichts von rammen gesagt lese es doch einfach noch mal
          Du hast immer noch nicht gesagt wie du auf die Zahlen kommst.
          Und der Trick ist nicht, dass der ganze Mech abgebremst wird, sondern dass er nur an einer Stelle abgebremst wird und die Trägheit der anderen Seite (in dem Fall oben oder unten) ihr übriges tut.

          Was glaubst du warum es wenig ratsam ist nur die Vorderbremse zu nutzen, wenn man mit dem Fahrrad mit volle Karacho den Berg runterrast. Das Vorderrad wird abgebremst, das hintere bewegt sich aber weiter, wodurch du einen Schönen Überschlag hinlegst.
          Ich möchte im Schlaf sterben wie mein Großvater, nicht schreiend und heulend wie sein Beifahrer im Wagen. - Will Shriner

          Kommentar


            Zitat von [OTG]Marauder Beitrag anzeigen
            Ich ERWARTE jetzt von dir das du eine Quelle nennst (Belegst) aus der diese zahlen stammen und wie du auf deine Berechnung kommst..
            ( ich habe die relevanten zahlen,der Mündungsgeschwindigkeiten und Geschossgewichte des Leo 2 parat (mit Link) und bin in der Lage deine Berechnungen zu überprüfen) also überasch uns mal mit FUNDIERTEM wissen (wenn du das kannst)
            Danke, das wollte ich auch gerade posten. Ich hätte nun gerne Zahlen, die man schön zerpflüggen kann! Und wenn sie stimmen sollten, dann sage ich mal Hut ab!

            Kommentar


              Zitat von nightwalker Beitrag anzeigen
              Hy Suthriel
              Wen ein Leopart2 einen 20t Mech mit einen DM63 trieft und die Panzerung hält wird sein Geschwindigkeit nach vorne um 0,696 Meter/s reduziert.
              Aha..die DM63 hat ein Geschoss gewicht von 8,45 kg im Penetrator und auf 3000m eine Mündungs/Einschlagsgeschwindikgeit von 1650 m/s...deine Formel zum von dir behaupteten will ich aber gerne mal sehen..
              Zitat von nightwalker Beitrag anzeigen
              ] Was wen er sich mit 14Meter/s bewegt kaum einen Einfluss hat. Wen er Steht und der Ausstemreflex ganze 0,4s dauert (was ultralangsam wäre) hätte er sich um 27,8 cm nach hinten bewegt was nicht gerade beeindrucken ist.
              auch diese Berechnung würde ich gerne sehen..ich denke aber das du die NICHT liefern kannst mein guter


              .
              EDIT (autom. Beitragszusammenführung) :

              [OTG]Marauder schrieb nach 5 Minuten und 17 Sekunden:

              Zitat von Jolly Beitrag anzeigen
              Danke, das wollte ich auch gerade posten. Ich hätte nun gerne Zahlen, die man schön zerpflüggen kann! Und wenn sie stimmen sollten, dann sage ich mal Hut ab!
              Zahlen hab ich grade gepostet..ich bin auf SEINE sehr gespannt

              und tante Edith sagt: Ich kenn nen Ingenieur,der berechnet tagtäglich zb. bis zu welcher Windgeschwindigkeit man noch einen 10m hohen Kran benutzen kann,bevor er droht umzufallen ...der wird mir dann berechnen wieviel es braucht ein 10 (oder mehr) hohes,auf 2 Beinen stehendes Ungetüm so anzuschubsen das es umfällt und ich glaube das wird wesentlich weniger sein als die 8,4Kg des penetrators,die mit über 1600m/s einschlagen

              Und um dem ganzen noch ein paar Zahlen zuzufügen: die Mündungsenergie der Leo2 Kanone liegt bei der "Alten" L/44 bei 9,8 Megajoule,bei der neuen L/55 bei über 13 MJ ...nach dem ja so sehr hier "beliebten" Energieerhaltungsgesetz,kann sich ja gerne jeder selber ausrechnen welche Energie ein "Wuchtgeschoss" mit ca.9kg,das mit (L/55) 1750m/s auf ein entweder festehendes oder sich gar gegen "Flugrichtung" des geschosses bewegendes Ziel ,ins Ziel bringt

              (ich glaub so langsam wirds richtig albern hier^^)
              Zuletzt geändert von [OTG]Marauder; 25.10.2010, 14:30. Grund: Antwort auf eigenen Beitrag innerhalb von 24 Stunden!
              .>ACHTUNG, freilaufender "Linker Gutmensch"! VORSICHT BISSIG!<

              Kommentar


                Da werden keine Rechnungen mehr kommen. Nach seinem letzten Beitrag wahr er noch lange genug online um alle unsere Forderungen nach seinem Rechnungsweg zu lesen. Wie wir sehen hat er nicht geantwortet.
                Ich möchte im Schlaf sterben wie mein Großvater, nicht schreiend und heulend wie sein Beifahrer im Wagen. - Will Shriner

                Kommentar


                  Habe es als Punktmassen gerechnet, als unelastischen Stoß und keine weiteren einwirkenden Kräfte.
                  Da alles andre ohne ein komplettes Modell von der Panzerung, Aufschlagwinkel echten Schwerpunkt, Elastizität der Aktoren, Stellung der Beine und Art der Füße keinen Sinn machen würde. Und das Ergebnis mit Seher hoher wahrscheinlich kleiner aus Fallen würde da die Beine ein Gegenkraft erzeugen würden.
                  Rechnung:
                  Die Summe der Impuls bei einen stoß muss der gleich sein wie vorher.
                  Wenn der Mecha steht, hat er zum Bezugssystem keinen Impuls. Nach dem Stoß muss V1*mp= V2*mp +V2*mm gelten. Daraus folgt V2=V1*mp/(mp+mm)
                  Die Masse eines DM63 ist 8,45 kg Geschwindigkeit des Penetrators 1650m/s erzeugen einen Impuls von 13942,5 kg*m/s. Wen man das dann durch die gesamt Masse von Mech und Projektil teilt kommt 0.696 m/s heraus.

                  Kommentar


                    Zitat von nightwalker Beitrag anzeigen
                    Habe es als Punktmassen gerechnet, als unelastischen Stoß und keine weiteren einwirkenden Kräfte.
                    Da alles andre ohne ein komplettes Modell von der Panzerung, Aufschlagwinkel echten Schwerpunkt, Elastizität der Aktoren, Stellung der Beine und Art der Füße keinen Sinn machen würde. Und das Ergebnis mit Seher hoher wahrscheinlich kleiner aus Fallen würde da die Beine ein Gegenkraft erzeugen würden.
                    Rechnung:
                    Die Summe der Impuls bei einen stoß muss der gleich sein wie vorher.
                    Wenn der Mecha steht, hat er zum Bezugssystem keinen Impuls. Nach dem Stoß muss V1*mp= V2*mp +V2*mm gelten. Daraus folgt V2=V1*mp/(mp+mm)
                    Die Masse eines DM63 ist 8,45 kg Geschwindigkeit des Penetrators 1650m/s erzeugen einen Impuls von 13942,5 kg*m/s. Wen man das dann durch die gesamt Masse von Mech und Projektil teilt kommt 0.696 m/s heraus.
                    Völliger Blödsinn..setzen 6.
                    deine "Formel" vergisst völlig die weitergehende Energieentfaltung auf die Gesamte Fläche nach auftreffen und Abgabe der Emitierten Bewegungsenergie..
                    Prüfung nicht bestanden,versuchen Sies nächstes jahr wieder
                    .>ACHTUNG, freilaufender "Linker Gutmensch"! VORSICHT BISSIG!<

                    Kommentar


                      [OTG]Marauder Die Rechnung ist korrekt, weiß ja nicht was man dir im Studium beigebracht hat aber du machst dich gerade ziemlich lächerlich.

                      Kommentar


                        Zitat von nightwalker Beitrag anzeigen
                        [OTG]Marauder Die Rechnung ist korrekt, weiß ja nicht was man dir im Studium beigebracht hat aber du machst dich gerade ziemlich lächerlich.
                        Um diesen Unfug von dir ad absurdum zu führen brauchts kein Studium (das ich übrigens nicht habe)

                        dir ist schon klar,das deine Formel nicht funktioniert,wenn man die entsprechenden Bewegungspunkte berücksichtigt??..


                        und dann berechne mal die besagte Bewegungsenergie für einen 10m hohen Körper mit mittigem Schwerpunkt und lege anschliessend dar,welche Kräfte dieser Körper aufbringen muss um die Schwerpunktverlagerung durch den Einschlag eines anderen Köpers (am äusseren Schwerpunktradius) im Gewicht von 8Kg mit 1800m/s zu kompensieren..

                        auf diese Berechnung bin ich sehr gespannt..
                        .>ACHTUNG, freilaufender "Linker Gutmensch"! VORSICHT BISSIG!<

                        Kommentar


                          Oh gut, jemand der einen Studierten Mechatroniker über Mechanik Physik und Generell Ingenieurwissenschaftlich mit seinen Schulwissen belehren will
                          Ich lache mich Tot

                          Kommentar


                            Zitat von nightwalker Beitrag anzeigen
                            Oh gut, jemand der einen Studierten Mechatroniker über Mechanik Physik und Generell Ingenieurwissenschaftlich mit seinen Schulwissen belehren will
                            Ich lache mich Tot
                            Anstatt zu lachen (was übrigens die meisten anderen hier über DICH tun) solltest du mir doch lieber meine Fragen beantworten und diese auch belegen,was du aber sinnigerweise nicht kannst

                            womit sich dann deine Glaubwürdigkeit "erledigt" hätte..aber ich muss zugeben das deine Ausführungen zwar "spinnert" aber dennoch kurzweilig amüsant sind
                            .>ACHTUNG, freilaufender "Linker Gutmensch"! VORSICHT BISSIG!<

                            Kommentar


                              Doch ich muss leider Lachen und ob das hier andere tun, ist egal, da das ganze meiner Privaten Unterhaltung dient, und nicht um irgend jemanden zu Überzeugen, was ja auch nicht geht.

                              Kommentar


                                Zitat von nightwalker Beitrag anzeigen
                                Oh gut, jemand der einen Studierten Mechatroniker über Mechanik Physik und Generell Ingenieurwissenschaftlich mit seinen Schulwissen belehren will
                                Ich lache mich Tot
                                Seit wann ist das ein Studienfach?

                                Und ein Mechatroniker kennt sich nicht zwangsläufig gut mit allgemeiner Physik aus. Mit ein paar Formel aus der Mechanik vielleicht. Jetzt verstehe ich wenigstens auch warum du ständig Rechnungen sehen willst. In jedem Teilbereich der Physik ist die Anzahl der Formeln recht begrenzt und es reicht bei vielem eine Erklärung wie es funktioniert. Nur die Mechanik braucht für jeden Mist eine eigene Formel.

                                Ich glaube aber nicht, dass man als Mechatroniker, viel mit Berechnungen zur Energie zu tun hat. Die sind aber das wichtigste bei Waffen.
                                Ich möchte im Schlaf sterben wie mein Großvater, nicht schreiend und heulend wie sein Beifahrer im Wagen. - Will Shriner

                                Kommentar

                                Lädt...
                                X